View Single Post
  #5  
Old 10-22-2006, 09:24 AM
elindauer elindauer is offline
Senior Member
 
Join Date: Jun 2003
Location: analyzing hand ranges
Posts: 2,966
Default thank you

ahhh.... Outstanding counter-example, thanks. I'm convinced there is no unique solution.


I'd like to hear more about your claim that a must not be greater than .2... can you explain this further? I don't see the conflict in the equations you mentioned.

edit: n/m. I'm still not sure about the equations you mentioned, but I have found a proof that a <= .2 (see response to PairTheBoard for this). Thanks! /edit

thanks,
eric
Reply With Quote